Practice Test 1 - Progress test 1 PDF

Title Practice Test 1 - Progress test 1
Author Vuong Huong Ly (k15 HL)
Course Mathematics Engineering
Institution FPT University
Pages 8
File Size 744.1 KB
File Type PDF
Total Downloads 72
Total Views 132

Summary

Progress test 1...


Description

27/5/2021

Practice Test 1

Home / My courses / MAE101 / *** Review for Test 1 *** / Practice Test 1

Started on Tuesday, 6 April 2021, 11:53 PM State Finished Completed on Wednesday, 7 April 2021, 12:08 AM Time taken 15 mins 57 secs Marks

20.00/20.00

Grade 10.00 out of 10.00 (100%)

Question 1

Find a, such that the function

has the domain [-5, 5]

Complete Mark 1.00 out of 1.00

Select one: a. a = - 25 b. c. a = 25 d. a = 5 e.

The correct answer is: a = 25

Question 2

Given that

and

Complete Mark 1.00 out of 1.00

Answer:

-2

The correct answer is: -2

, evaluate the limit.

27/5/2021

Practice Test 1

Question 3

Find the limit, if it exists.

Complete Mark 1.00 out of 1.00

Select one: a. -1 b. 1 c. 3 d. 5 e. 0 f. limit does not exist

The correct answer is: limit does not exist

Question 4

Find the number a such that the function

Complete Mark 1.00 out of 1.00

is continuous on its domain.

Select one: a. 1/2 b. 1/6 c. 1/8 d. None of the other choices is correct e. 1/4

The correct answer is: 1/8

27/5/2021

Practice Test 1

Question 6

The displacement of a particle on a vibrating string is given by the equation

Complete

where s is measured in centimeters and t in seconds. Find the velocity of the particle after t second

Mark 1.00 out of 1.00

Select one: a. b. c. d. e. None of the other choices is correct

The correct answer is:

Question 7

Find an equation of the tangent line to the curve.

Complete Mark 1.00 out of 1.00

at (4, 0.2)

Select one: a. y=(-1/100)(x-4)+0.2 b. None of the other choices is correct c. y=(-1/200)(x-4)+0.2 d. y=(1/100)(x-4)+0.2 e. y=(1/200)(x-4)+0.2

The correct answer is: y=(1/200)(x-4)+0.2

Question 8 Complete Mark 1.00 out of 1.00

Find the slope of the tangent line to the curve y3x+x2y = 10 at the point (1, 2)

27/5/2021

Practice Test 1

Question 9

Find a linear approximation for f(x)=x3 at x = 1.

Complete Mark 1.00 out of 1.00

Select one: a. None of the other choices is correct b. 3x+2 c. 2x-1 d. 3x-2 e. 2x+1

The correct answer is: 3x-2

Question

Find the absolute minimum value(s) of y=2x 2-20x+9 on the interval [0, 6].

10 Complete Mark 1.00 out of 1.00

Select one: a. -42 b. -38 c. -39 d. -41 e. -40

The correct answer is: -41

Question

Find the limit.

11 Complete Mark 1.00 out of 1.00

Select one: a. (a-b)/2

27/5/2021

Practice Test 1

Question

12

Find the minimum of the product of two numbers with the property that the is 7.

Complete Mark 1.00 out of 1.00

Select one: a. -3 b. 0 c. -7 d. None of the other choices is correct e. -4.25 f. -2.45

The correct answer is: -2.45

Question

13 Complete Mark 1.00 out of 1.00

Use Newton's method with the specified initial approximation x1 to find x 3, t root of the given equation. x3-3x+2=0, x1=3 (Give your answer to 4 decimal places)

Select one: a. None of the other choices is correct b. 1.3220 c. 1.6550 d. 1.7823 e. 1.4562

The correct answer is: 1.6550

Question

14 Complete Mark 1.00 out of 1 00

Find the antiderivative of the function.

f(x) = 8x1/7-10x 1/9

27/5/2021

Practice Test 1

Question

Use the Left-endpoint Rule with n = 5 to approximate the integral.

15 Complete Mark 1.00 out of 1.00

The choices are rounded to 3 decimal places.

Select one: a. 1.725 b. 0.766 c. 1.532 d. 0.863 e. None of the other choices is correct

The correct answer is: 1.725

Question

Express the limit as a definite integral on the given interval.

16 Complete Mark 1.00 out of 1.00

Select one: a.

b.

c. None of the other choices is correct

d.

27/5/2021

Practice Test 1

Question

Evaluate the indefinite integral

17 Complete Mark 1.00 out of 1.00

Select one: a. b. c. d.

The correct answer is:

Question

Evaluate the integral.

18 Complete Mark 1.00 out of 1.00

Select one: a. 7ex - 7xex + C b. 7ex - ex + C c. xex - 7ex + C d. 7xex - 7ex + C The correct answer is: 7xex - 7ex + C

Question

19

Use the Midpoint Rule with n = 3 steps to estimate the integral

Complete Mark 1.00 out of 1.00

x

2

2.5

3

f(x)

4

5

6

3.5 10

4

4.5

5

8

9

7

27/5/2021

Practice Test 1

Question

Determine whether the improper integral converges or diverges.

20 Complete Mark 1.00 out of 1.00

(i)

(ii)

Select one: a. Both converges b. Only (i) converges c. Both diverges d. Only (ii) converges

The correct answer is: Only (ii) converges

◄ LECTURE CHAPTER 3 (V2)

Jump to......


Similar Free PDFs